2
$\begingroup$

I recently came across the paper Les variétés de dimension 4 à signature non nulle dont la courbure est harmonique sont d’Einstein by Jean Pierre Bourguignon. What he shows in §8 is that the Weitzenböck curvature operator $\mathfrak{Ric}_\text{R}$ on $p$-forms is given by $$\mathfrak{Ric}_\text{R}(\omega)(X_1,\dots,X_p) = (\hat{\omega} \circ \hat{R_p})(X_1,\dots,X_p)$$ where $$R_p = \left(\tfrac{1}{2(p-1)}\text{Ric} \mathbin{\bigcirc\mspace{-19mu}\wedge\mspace{3mu}} \text{g} - \text{Rm}\right) \mathbin{\bigcirc\mspace{-19mu}\wedge\mspace{3mu}} \text{g}^{p-2}.$$ Here, I just can't figure out what he means by $\hat{\omega} \circ \hat{R_p}$.

Earlier in the paper he defines (in (2.11)) for a $C \in S^2\Lambda^{2}V$, interpreted as a self-adjoint map $\Lambda^{2}V \to \Lambda^{2}V$, and a form $\eta \in \Lambda^2(V)$, that $$\hat{C}(\eta) = \sum\limits_{i,j = 1}^n \eta(e_i,e_j)C(e_{i},e_{j})$$ for an orthonormal basis $(e_i)_{1 \le i \le n}$ of $V$.

I see how this can be generalized into: For a $C \in S^2\Lambda^{p}V$, interpreted as a self-adjoint map $\Lambda^{p}V \to \Lambda^{p}V$, and a form $\eta \in \Lambda^p(V)$, define $$\hat{C}(\eta) = \sum\limits_{i_1, \dots, i_p = 1}^n \eta(e_{i_1},\dots,e_{i_p})C(e_{i_1},\dots,e_{i_p})$$ for an orthonormal basis $(e_i)_{1 \le i \le n}$ of $V$. And I am thinking that this is what he means here for $C = R_k$. But this still doesn't answer what $\hat{\omega}$ would be.

I am thinking that $(\hat{\omega} \circ \hat{R_p})$ may just mean $\hat{R_p}(\omega)$; but why wouldn't he have written it like this then? Probably I am missing something central here..

The paper can be found here for free: Les variétés de dimension 4 à signature non nulle dont la courbure est harmonique sont d’Einstein by Jean Pierre Bourguignon

$\endgroup$

2 Answers 2

1
$\begingroup$

I think the idea is to think of $\hat{R}_p$ as a mapping from $\Lambda^pM$ to itself, and $\hat{\omega}$ as a mapping from $\Lambda^pM$ to $E$ (the vector bundle in which $\omega$ takes values), and then just compose these two operators (to get something $E$ valued in the end).

In the case where $E$ is the trivial (scalar) bundle, it works out to be $\hat{R}_p(\omega)$ since $R_p$ is symmetric.

$\endgroup$
2
  • $\begingroup$ Thank you very much! $\endgroup$
    – Mathy
    Sep 1, 2021 at 20:59
  • $\begingroup$ Hey Mr. Wong, I just reviewed your and my answer and I don't know anymore what I was thinking yesterday. Can you have a look at my edited answer below? $\endgroup$
    – Mathy
    Sep 2, 2021 at 14:08
1
$\begingroup$

This answer basically just writes out Willie Wong's.

In particular, the definition of $\hat{C}$ works for $\omega$ as well; if $\omega$ is a form with values in $E$, then $\hat{\omega}(\eta) = \sum\limits_{i_1, \dots, i_p = 1}^n \eta_{i_1,\dots,i_p}\omega_{i_1,\dots,i_p} \in E$. But then \begin{align} (\hat{\omega} \circ \hat{R_p})(X_1,\dots,X_p) &= \hat{\omega}(\hat{R_p}(X_1,\dots,X_p)) \\ &= \sum\limits_{i_1, \dots, i_p = 1}^n \omega_{i_1,\dots,i_p}R_p(X_1,\dots,X_p)(e_{i_1},\dots,e_{i_p}) \\ &= \sum\limits_{i_1, \dots, i_p = 1}^n \omega_{i_1,\dots,i_p}R_p(e_{i_1},\dots,e_{i_p})(X_1,\dots,X_p) \\ &= \hat{R_p}(\omega)(X_1,\dots,X_p). \end{align} At the second equality, we used that $R_p$ is symmetric; this is the case because the (generalized Kulkarni-Nomizu) product on the graded algebra $KV$ is commutative.

EDIT: Actually, I am not at all satisfied with this answer anymore. What did I do at the second equality yesterday? That doesn't make sense.

I will try again: Working directly with the definitions we have with $\eta = e_{j_1}\wedge\dots\wedge e_{j_p}$ that \begin{align*} (\hat{\omega} \circ \hat{R_p})(\eta) &= \hat{\omega}(\hat{R_p}(\eta)) \\ &= \hat{\omega}\left(\sum\limits_{i_1, \dots, i_p = 1}^n (e^{j_1} \wedge \dots \wedge e^{j_p})(e_{i_1}\wedge\dots\wedge e_{i_p}) \cdot (R_p)(e_{i_1},\dots,e_{i_p})\right) \\ &=\hat{\omega}\left(\sum\limits_{\tau \in S_p} (R_p)(e_{j_{\tau(1)}},\dots,e_{j_{\tau(p)}}) \right) \\ &= \sum\limits_{i_1, \dots, i_p = 1}^n \sum\limits_{\tau \in S_p} \omega_{i_1,\dots,i_p} (R_p)(e_{j_{\tau(1)}},\dots,e_{j_{\tau(p)}})(e_{i_1},\dots ,e_{i_p}). \end{align*} On the other hand, using the symmetry of $R_p$, we get \begin{align*} \hat{R}_p(\omega)(\eta) &= \left(\sum\limits_{i_1, \dots, i_p = 1}^n \omega_{i_1,\dots,i_p} \cdot (R_p)(e_{i_1},\dots ,e_{i_p}) \right)(\eta) \\ &= \sum\limits_{i_1, \dots, i_p = 1}^n \omega_{i_1,\dots,i_p} \cdot (R_p)(e_{i_1},\dots ,e_{i_p})(\eta) \\ &= \sum\limits_{i_1, \dots, i_p = 1}^n \omega_{i_1,\dots,i_p} \cdot (R_p)(\eta)(e_{i_1},\dots,e_{i_p}). \end{align*}

Actually, typing this out now I see that, using that $R_p$ is skew-symmetric in the first $p$ arguments, that the two terms are just off by a factor $\vert S_p \vert = p!$.

$\endgroup$
6
  • $\begingroup$ I don't quite follow your notation. But to me, the interpretation of $\hat{\omega}\circ \hat{R}_p(X_1, \ldots, X_p)$ would be the following: since $\hat{R}_p(X_1, \ldots, X_p)$ is a $p$-form, you can write it as $\sum \rho_{i_1i_2\dots i_p} e_{i_1} \wedge e_{i_2} \wedge \dots \wedge e_{i_p}$ for some basis. Then $\hat{\omega}(\hat{R}_p(X_1, \ldots, X_p)$ is given by $ \sum \omega(e_{i_1}\wedge\dots \wedge e_{i_p}) \rho_{i_1\dots i_p}$. $\endgroup$ Sep 2, 2021 at 17:22
  • $\begingroup$ Note that $\rho_{ij\dots k}$ is a real number and $\omega(e_i\dots e_{k})$ takes values in the fibre. Though honestly this is one case where I think abstract index notation can actually help make things more obvious. $\endgroup$ Sep 2, 2021 at 17:23
  • $\begingroup$ If $\omega$ is a vector valued $p$-form, we can use lower case latin indices for the base (pseudo Riemannian manifold) and upper case ones for the fibre, then $\hat{\omega}\circ \hat{R}_p$ is nothing more than $ \omega^A_{i_1\dots i_p} (R_p)^{i_1\dots i_p}_{j_1\dots j_p}$. $\endgroup$ Sep 2, 2021 at 17:25
  • $\begingroup$ I feel like you are saying that, if $(X_1,\dots,X_p) = (e_{j_1},\dots,e_{j_p})$, then the scalars $\rho_{i_1,\dots,i_p}$ are given by $\rho_{i_1,\dots,i_p} = R_p(e_{i_1},\dots,e_{i_p})(e_{j_1},\dots,e_{j_p})$; why would that be the case? If I compute it, I am again left with that thing times $p!$ which stems from the fact that we are summing all $i_1, \dots, i_p$ from $1$ to $n$ in the definition of $\hat{}$, and not only $i_1 < \dots < i_p$. $\endgroup$
    – Mathy
    Sep 3, 2021 at 8:19
  • $\begingroup$ Well... it depends a bit on how you define the wedge product, no? (Rather, how to you identify $\Lambda^pV$ as a subspace of $\otimes^p V$.) See also mathoverflow.net/questions/54343/… Factors of $p!$ almost always come up in these kinds of computations, and you just need to be consistent in what you do. $\endgroup$ Sep 3, 2021 at 13:11

Your Answer

By clicking “Post Your Answer”, you agree to our terms of service and acknowledge you have read our privacy policy.

Not the answer you're looking for? Browse other questions tagged or ask your own question.